User avatar
 
ohthatpatrick
Thanks Received: 3808
Atticus Finch
Atticus Finch
 
Posts: 4661
Joined: April 01st, 2011
 
This post thanked 1 time.
 
 

Re: Q24 - Had the party's economic theories been sound...

by ohthatpatrick Fri Dec 31, 1999 8:00 pm

Question Type:
Match the Flaw

Stimulus Breakdown:
Conclusion: Economic theories were off the mark.
Evidence: The inflation rate increased, and If the economic theories had been sound and their program had been implemented, inflation would have gone down.

Answer Anticipation:
Match the Flaw questions frequently test famous flaws, and since this one has conditional logic, we should look out for the Conditional Logic Flaw.

Indeed, the author messes up conditional logic, but it's not your typical illegal negation or reversal. Instead, the author tried to argue by contrapositive, but messes up how "and" and "or" work.

Given "A and B --> C",
the contrapositive is "~C --> ~A or ~B".

This author says "~C. Therefore, ~A".

A similarly flawed argument could sound like this: "Had Eddie scored a 160 and had he applied to law school, he would be in law school right now. But because he's not in law school, he must not have ever applied to law school." (How can we be so sure THAT'S why? Maybe he never got a 160)

We're looking for
A and B --> C.
~C.
Thus, ~A
(or ~B, doesn't matter which one)

Correct Answer:
C

Answer Choice Analysis:
(A) This conditional has "or", not "and", so let's move on.

(B) Looks tempting. "If wins and keeps promise, there will be a strike. Since there won't be a strike, won't keep promise." It's a little weird that its future tense. And the provided fact, the one that's supposed to trigger the contrapositive, is not actually a match for the contrapositive's trigger. There's a difference between "workers have promised not to strike" and "workers will not strike".

(C) Yes! "If had sold subsids and had bought new patent, stock price would have doubled. Stock price didn't double, so must not have sold subsids." (Same flaw as original: "How are we sure THAT's the culprit? Maybe it didn't buy the new patent.")

(D) This gives us "A and B --> C", but then the fact it provides is "C", whereas we're looking for "~C".

(E) This doesn't have an "and" in the trigger, so it won't be able to mimic the original flaw.

Takeaway/Pattern: Because the original flaw is conditional in nature, it's worth thinking about the stimulus in abstract terms. If we have a clear recipe, then we can make decisive eliminations without reading and processing the whole answer choice.
THE RECIPE
A and B -> C. ~C. Thus, ~A.

#officialexplanation
 
calebrosser
Thanks Received: 2
Vinny Gambini
Vinny Gambini
 
Posts: 6
Joined: July 13th, 2012
 
 
 

Q24 - Had the party's economic theories been sound...

by calebrosser Thu Oct 04, 2012 3:48 am

I had a difficult time with this question during my last PT, I narrowed it down to A and C. I had a gut feeling C was right, but couldn't quite explain why, and I still am not entirely convinced why it is right.

Could someone provide me with some guidance on this one?

Thanks
 
patrice.antoine
Thanks Received: 35
Atticus Finch
Atticus Finch
 
Posts: 111
Joined: November 02nd, 2010
 
 
 

Re: Q24 - Had the party's economic theories been sound...

by patrice.antoine Thu Oct 04, 2012 4:13 pm

I'll take a bite:

Stimulus Set Up:

Key Symbols
SET - Sound Economic Theories
SIP - Succeeding Implementing Program
LI - Lower Inflation

So the stimulus is saying:

Premise 1: If SET & SIP --> LI
Premise 2: ~LI
________________
Conclusion: ~ SET

That's the general structure of the argument.

Answer choice (A) Set Up:

Key Symbols
IVI - Inhabited Valley Invasion
DCC - Dramatic Climate Change
CVA - Change in Valley Architecture

Premise 1: If IVI --> CVA
Premise 2: If DCC --> CVA
Premise 3: ~ CVA
__________________
Conclusion: ~IVI

Answer choice (C) set up:

Key Symbols
SSS - Success Selling Subsidiaries
CP - Cash Purchase
SPD - Stock Price Doubled

Premise 1: If SSS & CP --> SP
Premise 2: ~SPD
_____________________
Conclusion: ~SSS

*********************************

Do you see the difference here? In answer choice (A) there are THREE premises instead of the TWO presented in the stimulus and in answer choice (C). Answer choice (A) presents two "if...then" statements, causing there to be an additional premise. This does not follow the paralleled reasoning presented in the stimulus.
 
hychu3
Thanks Received: 3
Vinny Gambini
Vinny Gambini
 
Posts: 20
Joined: June 01st, 2013
 
 
 

Re: Q24 - Had the party's economic theories been sound...

by hychu3 Wed Nov 20, 2013 8:54 pm

Hi,

I must disagree with the previous poster's explanation of this question.

Here, we're asked to match the flawed reasoning, not the overall reasoning. In these types of questions, unlike other parallel reasoning questions, the emphasis is placed on the flaw itself. Other parts of the argument are insignificant as long as they do not interfere with the flaw in question.

The flaw in this stimulus is invalid contrapositive. The stimulus says:

(1) ETS and SIP -> IRL

Therefore,

(2) ~IRL -> ~ETS.

The last line is an invalid inference. That is, (1) does not imply (2). Here is why. By taking contrapositive of (1), we have

~IRL -> ~(ETS and SIP) = ~ETS or ~SIP.

Now, as we can see from the last line, if ~IRL, we still do not know if ~ETS. We know either ~ETS or ~SIP (or perhaps both), but we do not know for sure which one it is. For example, perhaps the party's economic theory was indeed sound, but its implementation failed.

(C) displays this flaw. Who knows, perhaps the company did sell its subsidiaries but did not purchase the new patent.

(A) actually has no flaw. Its conclusion can be logically derived from taking contrapositive of the first sentence.

(B) is a lousy argument, but it actually does not display any logical flaw. Read it carefully: it's just a bunch of speculations about what other people think.

(D) confuses the necessary and sufficient conditions. Not the same flaw.

(E) is another lousy argument, but, like (B), it does not display any logical flaw. This is because we do not know how "worth the investment" and "improvement on the old" are related. If these two mean the same thing (they actually seem to be), this is actually a logically valid argument.

Hope this helps. :)
User avatar
 
ohthatpatrick
Thanks Received: 3808
Atticus Finch
Atticus Finch
 
Posts: 4661
Joined: April 01st, 2011
 
This post thanked 1 time.
 
 

Re: Q24 - Had the party's economic theories been sound...

by ohthatpatrick Thu Nov 21, 2013 6:43 pm

Great response! I agree that on Match the Flaw, we should be primarily attuned to diagnosing the original flaw. We don't need to (or necessarily want to) be as structurally picky when it comes to finding our correct answer.

That said, most of the time the correct answer does still mimic the structure and flaw of the original, so looking for structural matches/mismatches can still be an efficient way of screening out unlikely answers. But the previous poster is correct to say that the correct answer technically only has to embody the same flaw; it doesn't have to have identical structure.

The only tweak I would make to the last post is to say that (B) and (E) do indeed have flaws. I think the poster just meant that (B) and (E) don't have conditional logic flaws such as the one we're seeking from the original.

(E) is flawed due to its "not worth the investment" = "no improvement" language shift (reasonable as it is).

(B) is flawed because the author attempts to derive a true statement about workers based on a conditional rule that is not actually a rule, but rather the speculative thoughts of many people's fears.

Great write-up!!
 
hychu3
Thanks Received: 3
Vinny Gambini
Vinny Gambini
 
Posts: 20
Joined: June 01st, 2013
 
 
 

Re: Q24 - Had the party's economic theories been sound...

by hychu3 Sun Dec 01, 2013 10:47 pm

Thanks for pointing that out. I should've said "flaws in conditional reasoning" instead of just "flaws." (B) and (E) are definitely flawed.
 
Sweetangel
Thanks Received: 0
Vinny Gambini
Vinny Gambini
 
Posts: 17
Joined: April 30th, 2014
 
 
 

Re: Q24 - Had the party's economic theories been sound...

by Sweetangel Fri Nov 21, 2014 9:07 pm

I don't agree that C matches the flaw. I don't think we should be looking for an equivalent of ~IRL (which could be just an unchaned IR); the stimulus goes beyond that by citing an increase in the inflation rate. I don't think any of the answer choices resemble this, other than E, which we've established is not a perfect match for other reasons. Anybody feel me??
 
dhlim3
Thanks Received: 4
Jackie Chiles
Jackie Chiles
 
Posts: 34
Joined: January 19th, 2015
 
 
 

Re: Q24 - Had the party's economic theories been sound...

by dhlim3 Thu Sep 17, 2015 6:33 am

hychu3 Wrote:Hi,

I must disagree with the previous poster's explanation of this question.

Here, we're asked to match the flawed reasoning, not the overall reasoning. In these types of questions, unlike other parallel reasoning questions, the emphasis is placed on the flaw itself. Other parts of the argument are insignificant as long as they do not interfere with the flaw in question.

The flaw in this stimulus is invalid contrapositive. The stimulus says:

(1) ETS and SIP -> IRL

Therefore,

(2) ~IRL -> ~ETS.

The last line is an invalid inference. That is, (1) does not imply (2). Here is why. By taking contrapositive of (1), we have

~IRL -> ~(ETS and SIP) = ~ETS or ~SIP.

Now, as we can see from the last line, if ~IRL, we still do not know if ~ETS. We know either ~ETS or ~SIP (or perhaps both), but we do not know for sure which one it is. For example, perhaps the party's economic theory was indeed sound, but its implementation failed.

(C) displays this flaw. Who knows, perhaps the company did sell its subsidiaries but did not purchase the new patent.

(A) actually has no flaw. Its conclusion can be logically derived from taking contrapositive of the first sentence.

(B) is a lousy argument, but it actually does not display any logical flaw. Read it carefully: it's just a bunch of speculations about what other people think.

(D) confuses the necessary and sufficient conditions. Not the same flaw.

(E) is another lousy argument, but, like (B), it does not display any logical flaw. This is because we do not know how "worth the investment" and "improvement on the old" are related. If these two mean the same thing (they actually seem to be), this is actually a logically valid argument.

Hope this helps. :)



Agree with most of this except A) actually does have a flaw.

The contrapositive of "OR" conditional statements become an "AND" statement (A OR B -> C, therefore, ~C -> ~A AND ~B). Answer choice A does not apply this rule. The correct conclusion for A) should have been, "Thus, the valley people must not have been invaded at any time during their stay AND there must not had been a dramatic climatic change."